LSAT and Law School Admissions Forum

Get expert LSAT preparation and law school admissions advice from PowerScore Test Preparation.

 Adam Tyson
PowerScore Staff
  • PowerScore Staff
  • Posts: 5153
  • Joined: Apr 14, 2011
|
#36268
I'll give it a shot, although others here have already done yeoman's work on the diagram and also approached it in a more holistic, non-diagrammatic way. Clearly, based on how many people are confused by this question (we're up to something like 60 posts about it now!) this is one of the hardest and most confusing LR stimuli floating around out there, so you are not alone!

The first sentence uses a classic necessary condition indicator, "except". Using our "Unless Equation" (which applies to the words unless, except, until, and without), we get this diagram:

wrong to restrict a liberty :arrow: failing to restrict allows harm

(I would read that as "if it isn't wrong to restrict a liberty, then failing to restrict that liberty would allow some harm")

The last sentence gives us nothing conditional to diagram, in my opinion, but merely categorizes two things: publishing is a liberty (so it falls under the umbrella of the sufficient condition) and offending someone isn't harmful (so it does NOT fall under the umbrella of the necessary condition - it's not harmful).

The contrapositive of our diagram is:

failing to restrict allows harm :arrow: wrong to restrict a liberty

Now let's put that all together: if failing to restrict a liberty does not allow some harm, then it would be wrong to restrict it. In other words, if it doesn't hurt anyone, government should leave it alone. That's answer A! Literature that is only offensive is not harmful, and so restricting it would be wrong.

I hope that helps!
 lathlee
  • Posts: 652
  • Joined: Apr 01, 2016
|
#39019
Hi. actually I do not have a probelm with anything else but the wording of the answer a)

offensive :arrow: wrong for gov to restrict liberty or (publication)

since the term is decorated with the wording of only shouldn't offensive become the necessary condition if conditional relaotinship to be formed

the choice a) it is not right for the government to restrict the publication of literature that is only offensive. I just don't get how this senetence can create any powerful enough conditional relationship.
 James Finch
PowerScore Staff
  • PowerScore Staff
  • Posts: 943
  • Joined: Sep 06, 2017
|
#39586
Hi Lathlee,

As Adam noted above, the relationship we have from the first sentence in the stimulus is:

wrong to restrict a liberty :arrow: failing to restrict allows harm

or the contrapositive:

failing to restrict allows harm :arrow: wrong to restrict a liberty

The second sentence then gives us:

offend :arrow: harm

Put them together and we get:

offend :arrow: harm :arrow: wrong to restrict a liberty

or

offend :arrow: wrong to restrict a liberty

Answer choice (A) gives us exactly that last combination, that "it is not right for the government to restrict the publication of literature that is only offensive."

Hope this clears things up!
 m_mey
  • Posts: 3
  • Joined: Oct 12, 2017
|
#40489
I understand why A is right, but I'm not entirely sure why B is wrong. When I look at it, it looks like B is a conditional statement that restates what is in the first sentence of the stimulus. The stimulus states that, it is wrong for the government to restrict rights EXCEPT in cases when failure to do so would allow individuals to cause harm. B does not state the exception, but wouldn't the clause that says "when failing to do so would allow individuals to cause harm," essentially mean the exceptional instances similar to the statement in the stimulus? This is an assumption and I don't know if this is really getting at why B wrong, but I don't fully understand why B is wrong.
 nicholaspavic
PowerScore Staff
  • PowerScore Staff
  • Posts: 271
  • Joined: Jun 12, 2017
|
#40511
Hi mey!

Thanks for the message and welcome to the Forum!

This is always a tricky question for students in their review. But let's take a close look at Answer Choice (B) and give it a good analysis.

Answer choice (B) uses conditional language, as the word "when" is a sufficient condition indicator ("when" functions similarly to "if"). So when we diagram (B), it will look like this:

..... Harm :arrow: Not Wrong Restrict Liberty

That's a loose Mistaken Reversal of the statement that we have in the stimulus because the stimulus' first sentence isn't "perfectly phrased" conditional reasoning (the word "perhaps" softens the language). But when we diagram this soft statement, it looks like this:

..... Not Wrong Restrict Liberty :arrow: Harm

So with this stimulus' statement, knowing that it was not wrong (or permissible) to actually restrict individual liberty would tell you that doing so must allow those individuals to cause harm. Put another way, if liberty is going to be restricted, you know the one possible reason to restrict it--the exception to freedom: cause harm--must occur.

So in that sense Answer choice (B) does reverse the ideas from the first sentence, and would generally fall under the category of a Mistaken Reversal as discussed in our course.

Please let us know if that helps and welcome again!
 Blueballoon5%
  • Posts: 156
  • Joined: Jul 13, 2015
|
#44602
Hello! Could someone explain to be how answer choice A is translated to a conditional statement. The answer choice a reads: "It is not right for the government to restrict the publication of literature that is only offensive." What are the sufficient/necessary indicators in this sentence?

I thought that "only" was the necessary indicator (and thus, "offensive" is the necessary condition). However, based on the explanations in this forum, it seems like the "offensive" part of the sentence is the sufficient condition.
 elewis10
  • Posts: 21
  • Joined: Sep 02, 2017
|
#44842
I see why A is correct. I picked B, though, and do not see why it is wrong. Is it the mistaken reversal? If so, why is that? Thank you!!
 lilmissunshine
  • Posts: 94
  • Joined: Jun 07, 2018
|
#46628
Hello,

All the previous explanations are very helpful. However, I was wondering why we could assume "not right" = "wrong" in (A) and (E). "not right" could also be neutral right? It seems like the answer choices confuse polar opposite with logic opposite. Could someone explain that to me?

Thanks a lot!
 BostonLawGuy
  • Posts: 52
  • Joined: Jul 13, 2018
|
#48944
After all the past posts on this question, I'm still a bit confused. This question demands conditional diagramming and what has already been diagramed is a bit confusing. Specifically, there does not seem to be any evidence in the stimulus about "preventing" harm, only causing or not causing harm. There is nothing mentioning "preventing harm" in the stimulus and I think that is where my confusion lies since that term is the one that links the premises.

My diagram seems to make sense enabling easier linkage but I would like confirmation if it's actually accurate and if it is, am I reading the contrapositive correctly? I am using CAPS to show what and how I linked the premises.

It's not wrong to restrict liberty :arrow: (fi not to do so, would) CAUSE HARM
Publishing :arrow: liberty
Offend :arrow: not CAUSE HARM

STEP 2: CONDITIONAL LINKAGE (using contrapositive of offend :arrow: not cause harm)

It is not wrong to restrict liberties :arrow: CAUSE HARM :arrow: /offend

Reading the contrapositive of this statement is an exact match for the correct answer A. Did I arrive at the right conclusion using correct conditional logic linkage?
 James Finch
PowerScore Staff
  • PowerScore Staff
  • Posts: 943
  • Joined: Sep 06, 2017
|
#48979
Hi BLG,

This question actually only contains one conditional statment, in the first sentence:

Wrong to Restrict Liberty (WRL) :arrow: Cause Harm (CH)

and its contrapositive:

CH :arrow: WRL

Then it gives us specific examples that would fall under the general conditions given:

Publish = Liberty

Offend = Cause Harm

From this, we can see we have to apply the contrapositive to the situation, which gives us:

CHOffend :arrow: WRLPublish

Meaning that it would be wrong to restrict publication of offensive material. Answer choice (A) reflects this, making it the correct answer.

Hope this helps!
  • 1
  • 4
  • 5
  • 6
  • 7
  • 8

Get the most out of your LSAT Prep Plus subscription.

Analyze and track your performance with our Testing and Analytics Package.